How to prove that the definition of exterior product of differential forms is not ambiguous?












0












$begingroup$


In page 91 of book A Visual Introduction to Differential Forms and Calculus on Manifolds the exterior product of two differential forms $alpha in bigwedge^{r}(mathbb{R}^n)$ and $beta in bigwedge^{s}(mathbb{R}^n)$ is definide by
$$
alpha wedge beta = left( sum_{I}a_I dx^Iright) wedge left( sum_{J}b_J dx^Jright)=sum_{I}sum_{J}a_Ib_J dx^Iwedge dx^J
hspace{2cm}(ast)
$$

where $alpha =sum_{I}a_I dx^I$ and $beta =sum_{J}b_J dx^J$. Here ${dx^I}_{I}$ is the basis of $bigwedge^{r}(mathbb{R}^n)$ that we get ( by wedge product $dx^I=dx^{i_1}wedge ldotswedge dx^{i_k}$) from basis ${dx^1, ldots, dx^n}$ of $(mathbb{R}^n)^ast$ dual of canonical basis ${e_1,ldots,e_n}$ of $mathbb{R}^n$. In the same way, ${dx^J}_{J}$ is the basis of $bigwedge^{s}(mathbb{R}^n)$ that we get ( by wedge product $dx^J=dx^{j_1}wedge ldotswedge dx^{j_ell}$) from basis ${dx^1, ldots, dx^n}$ of $(mathbb{R}^n)^ast$ dual of canonical basis ${e_1,ldots,e_n}$ of $mathbb{R}^n$.



Do Carmo in his book Differential Forms and Applications makes the same definition of exterior product of differential forms.



Now, let's set any base in $ mathbb{R}^n$, say ${y_1,ldots,y_n}$. Let ${dy^I}_{I}$ is the base of $bigwedge^{r}(mathbb{R}^n)$ that we get ( by wedge product $dy^I=dy^{i_1}wedge ldotswedge dy^{i_k}$) from basis ${dy^1, ldots, dy^n}$ of $(mathbb{R}^n)^ast$ dual of basis ${y_1,ldots,y_n}$ of $mathbb{R}^n$. In the same way, let ${dy^J}_{J}$ the basis of $bigwedge^{s}(mathbb{R}^n)$ that we get ( by wedge product $dy^J=dy^{j_1}wedge ldotswedge dy^{j_ell}$) from basis ${dy^1, ldots, dy^n}$ of $(mathbb{R}^n)^ast$ dual of canonical basis ${y_1,ldots,y_n}$ of $mathbb{R}^n$.



Since ${dy^I}_{I}$ and ${dy^J}_{J}$ are basis of $bigwedge^{r}(mathbb{R}^n)$ and $bigwedge^{s}(mathbb{R}^n)$ respectively we can rewrite the differential forms $alpha$ and $beta$ in terms of these basis as
$$
alpha = sum_{I}g_I dy^I hspace{1cm} mbox{and}hspace{1cm} beta=sum_{J} h_{J} dy^J.
$$



It is not clear, at least for me, that
$$
alphawedge beta = left( sum_{I}g_I dy^I right) wedge left( sum_{J} h_{J} dy^J right)= sum_{I}sum_{J} g_Ih_J dy^I wedge dy^J
$$

will be the same differential form that we get in $(ast)$.




Question. How to prove that the definition of exterior product of differential forms is not ambiguous? That is, how to prove that
$$
left( sum_{I}a_I dx^Iright) wedge left( sum_{J}b_J dx^Jright)
=
left( sum_{I}g_I dy^I right) wedge left( sum_{J} h_{J} dy^Jright) mbox{ ? }
$$




Honestly, I have no idea how to proceed with this problem.










share|cite|improve this question











$endgroup$












  • $begingroup$
    This is just because of the algebraic properties of $wedge$. Have you tried writing it out explicitly with $alpha = dx^1$ and $beta=dx^2$ for starters?
    $endgroup$
    – Ted Shifrin
    Jan 11 at 18:45










  • $begingroup$
    @TedShifrin In this case supose $alpha, beta bigwedge^1(mathbb{R}^2)$. We will have $alpha= dx^1=g(y_1,y_2)dy^1$, $beta= dx^2=h(y_1,y_2)dy^2$, $alphawedge beta(e_1,e_2)=1$ and $alphawedge beta g(y)h(y)det begin{pmatrix} dy^1(e_1) & dy^1(e_2) \ dy^2(e_1) & dy^2(e_2) end{pmatrix}$. But how can I use this?
    $endgroup$
    – MathOverview
    Jan 11 at 20:12












  • $begingroup$
    First, it's just about pointwise linear algebra. Second, don't do it by evaluating. Write $dy^i = sum a^i_j dx^j$. (In your set-up, you did the reverse. It doesn't matter.)
    $endgroup$
    – Ted Shifrin
    Jan 11 at 21:30










  • $begingroup$
    @TedShifrin It would be something of the type $dy^{I}wedge dy^{J}=dy^{i_1}wedge ldots wedge dy^{i_k}wedge dy^{j_1}wedge ldots wedge dy^{j_ell}=left( sum_{u=1}^{n} a_u^{i_1} dx^u right)wedge ldotswedge left( sum_{u=1}^{n} a_u^{i_k} dx^u right)wedge left( sum_{v=1}^{n} a_v^{j_1} dx^v right) wedge ldotswedge left( sum_{v=1}^{n} a_v^{j_ell} dx^v right);;;;?$
    $endgroup$
    – MathOverview
    Jan 11 at 21:50


















0












$begingroup$


In page 91 of book A Visual Introduction to Differential Forms and Calculus on Manifolds the exterior product of two differential forms $alpha in bigwedge^{r}(mathbb{R}^n)$ and $beta in bigwedge^{s}(mathbb{R}^n)$ is definide by
$$
alpha wedge beta = left( sum_{I}a_I dx^Iright) wedge left( sum_{J}b_J dx^Jright)=sum_{I}sum_{J}a_Ib_J dx^Iwedge dx^J
hspace{2cm}(ast)
$$

where $alpha =sum_{I}a_I dx^I$ and $beta =sum_{J}b_J dx^J$. Here ${dx^I}_{I}$ is the basis of $bigwedge^{r}(mathbb{R}^n)$ that we get ( by wedge product $dx^I=dx^{i_1}wedge ldotswedge dx^{i_k}$) from basis ${dx^1, ldots, dx^n}$ of $(mathbb{R}^n)^ast$ dual of canonical basis ${e_1,ldots,e_n}$ of $mathbb{R}^n$. In the same way, ${dx^J}_{J}$ is the basis of $bigwedge^{s}(mathbb{R}^n)$ that we get ( by wedge product $dx^J=dx^{j_1}wedge ldotswedge dx^{j_ell}$) from basis ${dx^1, ldots, dx^n}$ of $(mathbb{R}^n)^ast$ dual of canonical basis ${e_1,ldots,e_n}$ of $mathbb{R}^n$.



Do Carmo in his book Differential Forms and Applications makes the same definition of exterior product of differential forms.



Now, let's set any base in $ mathbb{R}^n$, say ${y_1,ldots,y_n}$. Let ${dy^I}_{I}$ is the base of $bigwedge^{r}(mathbb{R}^n)$ that we get ( by wedge product $dy^I=dy^{i_1}wedge ldotswedge dy^{i_k}$) from basis ${dy^1, ldots, dy^n}$ of $(mathbb{R}^n)^ast$ dual of basis ${y_1,ldots,y_n}$ of $mathbb{R}^n$. In the same way, let ${dy^J}_{J}$ the basis of $bigwedge^{s}(mathbb{R}^n)$ that we get ( by wedge product $dy^J=dy^{j_1}wedge ldotswedge dy^{j_ell}$) from basis ${dy^1, ldots, dy^n}$ of $(mathbb{R}^n)^ast$ dual of canonical basis ${y_1,ldots,y_n}$ of $mathbb{R}^n$.



Since ${dy^I}_{I}$ and ${dy^J}_{J}$ are basis of $bigwedge^{r}(mathbb{R}^n)$ and $bigwedge^{s}(mathbb{R}^n)$ respectively we can rewrite the differential forms $alpha$ and $beta$ in terms of these basis as
$$
alpha = sum_{I}g_I dy^I hspace{1cm} mbox{and}hspace{1cm} beta=sum_{J} h_{J} dy^J.
$$



It is not clear, at least for me, that
$$
alphawedge beta = left( sum_{I}g_I dy^I right) wedge left( sum_{J} h_{J} dy^J right)= sum_{I}sum_{J} g_Ih_J dy^I wedge dy^J
$$

will be the same differential form that we get in $(ast)$.




Question. How to prove that the definition of exterior product of differential forms is not ambiguous? That is, how to prove that
$$
left( sum_{I}a_I dx^Iright) wedge left( sum_{J}b_J dx^Jright)
=
left( sum_{I}g_I dy^I right) wedge left( sum_{J} h_{J} dy^Jright) mbox{ ? }
$$




Honestly, I have no idea how to proceed with this problem.










share|cite|improve this question











$endgroup$












  • $begingroup$
    This is just because of the algebraic properties of $wedge$. Have you tried writing it out explicitly with $alpha = dx^1$ and $beta=dx^2$ for starters?
    $endgroup$
    – Ted Shifrin
    Jan 11 at 18:45










  • $begingroup$
    @TedShifrin In this case supose $alpha, beta bigwedge^1(mathbb{R}^2)$. We will have $alpha= dx^1=g(y_1,y_2)dy^1$, $beta= dx^2=h(y_1,y_2)dy^2$, $alphawedge beta(e_1,e_2)=1$ and $alphawedge beta g(y)h(y)det begin{pmatrix} dy^1(e_1) & dy^1(e_2) \ dy^2(e_1) & dy^2(e_2) end{pmatrix}$. But how can I use this?
    $endgroup$
    – MathOverview
    Jan 11 at 20:12












  • $begingroup$
    First, it's just about pointwise linear algebra. Second, don't do it by evaluating. Write $dy^i = sum a^i_j dx^j$. (In your set-up, you did the reverse. It doesn't matter.)
    $endgroup$
    – Ted Shifrin
    Jan 11 at 21:30










  • $begingroup$
    @TedShifrin It would be something of the type $dy^{I}wedge dy^{J}=dy^{i_1}wedge ldots wedge dy^{i_k}wedge dy^{j_1}wedge ldots wedge dy^{j_ell}=left( sum_{u=1}^{n} a_u^{i_1} dx^u right)wedge ldotswedge left( sum_{u=1}^{n} a_u^{i_k} dx^u right)wedge left( sum_{v=1}^{n} a_v^{j_1} dx^v right) wedge ldotswedge left( sum_{v=1}^{n} a_v^{j_ell} dx^v right);;;;?$
    $endgroup$
    – MathOverview
    Jan 11 at 21:50
















0












0








0





$begingroup$


In page 91 of book A Visual Introduction to Differential Forms and Calculus on Manifolds the exterior product of two differential forms $alpha in bigwedge^{r}(mathbb{R}^n)$ and $beta in bigwedge^{s}(mathbb{R}^n)$ is definide by
$$
alpha wedge beta = left( sum_{I}a_I dx^Iright) wedge left( sum_{J}b_J dx^Jright)=sum_{I}sum_{J}a_Ib_J dx^Iwedge dx^J
hspace{2cm}(ast)
$$

where $alpha =sum_{I}a_I dx^I$ and $beta =sum_{J}b_J dx^J$. Here ${dx^I}_{I}$ is the basis of $bigwedge^{r}(mathbb{R}^n)$ that we get ( by wedge product $dx^I=dx^{i_1}wedge ldotswedge dx^{i_k}$) from basis ${dx^1, ldots, dx^n}$ of $(mathbb{R}^n)^ast$ dual of canonical basis ${e_1,ldots,e_n}$ of $mathbb{R}^n$. In the same way, ${dx^J}_{J}$ is the basis of $bigwedge^{s}(mathbb{R}^n)$ that we get ( by wedge product $dx^J=dx^{j_1}wedge ldotswedge dx^{j_ell}$) from basis ${dx^1, ldots, dx^n}$ of $(mathbb{R}^n)^ast$ dual of canonical basis ${e_1,ldots,e_n}$ of $mathbb{R}^n$.



Do Carmo in his book Differential Forms and Applications makes the same definition of exterior product of differential forms.



Now, let's set any base in $ mathbb{R}^n$, say ${y_1,ldots,y_n}$. Let ${dy^I}_{I}$ is the base of $bigwedge^{r}(mathbb{R}^n)$ that we get ( by wedge product $dy^I=dy^{i_1}wedge ldotswedge dy^{i_k}$) from basis ${dy^1, ldots, dy^n}$ of $(mathbb{R}^n)^ast$ dual of basis ${y_1,ldots,y_n}$ of $mathbb{R}^n$. In the same way, let ${dy^J}_{J}$ the basis of $bigwedge^{s}(mathbb{R}^n)$ that we get ( by wedge product $dy^J=dy^{j_1}wedge ldotswedge dy^{j_ell}$) from basis ${dy^1, ldots, dy^n}$ of $(mathbb{R}^n)^ast$ dual of canonical basis ${y_1,ldots,y_n}$ of $mathbb{R}^n$.



Since ${dy^I}_{I}$ and ${dy^J}_{J}$ are basis of $bigwedge^{r}(mathbb{R}^n)$ and $bigwedge^{s}(mathbb{R}^n)$ respectively we can rewrite the differential forms $alpha$ and $beta$ in terms of these basis as
$$
alpha = sum_{I}g_I dy^I hspace{1cm} mbox{and}hspace{1cm} beta=sum_{J} h_{J} dy^J.
$$



It is not clear, at least for me, that
$$
alphawedge beta = left( sum_{I}g_I dy^I right) wedge left( sum_{J} h_{J} dy^J right)= sum_{I}sum_{J} g_Ih_J dy^I wedge dy^J
$$

will be the same differential form that we get in $(ast)$.




Question. How to prove that the definition of exterior product of differential forms is not ambiguous? That is, how to prove that
$$
left( sum_{I}a_I dx^Iright) wedge left( sum_{J}b_J dx^Jright)
=
left( sum_{I}g_I dy^I right) wedge left( sum_{J} h_{J} dy^Jright) mbox{ ? }
$$




Honestly, I have no idea how to proceed with this problem.










share|cite|improve this question











$endgroup$




In page 91 of book A Visual Introduction to Differential Forms and Calculus on Manifolds the exterior product of two differential forms $alpha in bigwedge^{r}(mathbb{R}^n)$ and $beta in bigwedge^{s}(mathbb{R}^n)$ is definide by
$$
alpha wedge beta = left( sum_{I}a_I dx^Iright) wedge left( sum_{J}b_J dx^Jright)=sum_{I}sum_{J}a_Ib_J dx^Iwedge dx^J
hspace{2cm}(ast)
$$

where $alpha =sum_{I}a_I dx^I$ and $beta =sum_{J}b_J dx^J$. Here ${dx^I}_{I}$ is the basis of $bigwedge^{r}(mathbb{R}^n)$ that we get ( by wedge product $dx^I=dx^{i_1}wedge ldotswedge dx^{i_k}$) from basis ${dx^1, ldots, dx^n}$ of $(mathbb{R}^n)^ast$ dual of canonical basis ${e_1,ldots,e_n}$ of $mathbb{R}^n$. In the same way, ${dx^J}_{J}$ is the basis of $bigwedge^{s}(mathbb{R}^n)$ that we get ( by wedge product $dx^J=dx^{j_1}wedge ldotswedge dx^{j_ell}$) from basis ${dx^1, ldots, dx^n}$ of $(mathbb{R}^n)^ast$ dual of canonical basis ${e_1,ldots,e_n}$ of $mathbb{R}^n$.



Do Carmo in his book Differential Forms and Applications makes the same definition of exterior product of differential forms.



Now, let's set any base in $ mathbb{R}^n$, say ${y_1,ldots,y_n}$. Let ${dy^I}_{I}$ is the base of $bigwedge^{r}(mathbb{R}^n)$ that we get ( by wedge product $dy^I=dy^{i_1}wedge ldotswedge dy^{i_k}$) from basis ${dy^1, ldots, dy^n}$ of $(mathbb{R}^n)^ast$ dual of basis ${y_1,ldots,y_n}$ of $mathbb{R}^n$. In the same way, let ${dy^J}_{J}$ the basis of $bigwedge^{s}(mathbb{R}^n)$ that we get ( by wedge product $dy^J=dy^{j_1}wedge ldotswedge dy^{j_ell}$) from basis ${dy^1, ldots, dy^n}$ of $(mathbb{R}^n)^ast$ dual of canonical basis ${y_1,ldots,y_n}$ of $mathbb{R}^n$.



Since ${dy^I}_{I}$ and ${dy^J}_{J}$ are basis of $bigwedge^{r}(mathbb{R}^n)$ and $bigwedge^{s}(mathbb{R}^n)$ respectively we can rewrite the differential forms $alpha$ and $beta$ in terms of these basis as
$$
alpha = sum_{I}g_I dy^I hspace{1cm} mbox{and}hspace{1cm} beta=sum_{J} h_{J} dy^J.
$$



It is not clear, at least for me, that
$$
alphawedge beta = left( sum_{I}g_I dy^I right) wedge left( sum_{J} h_{J} dy^J right)= sum_{I}sum_{J} g_Ih_J dy^I wedge dy^J
$$

will be the same differential form that we get in $(ast)$.




Question. How to prove that the definition of exterior product of differential forms is not ambiguous? That is, how to prove that
$$
left( sum_{I}a_I dx^Iright) wedge left( sum_{J}b_J dx^Jright)
=
left( sum_{I}g_I dy^I right) wedge left( sum_{J} h_{J} dy^Jright) mbox{ ? }
$$




Honestly, I have no idea how to proceed with this problem.







differential-forms multilinear-algebra






share|cite|improve this question















share|cite|improve this question













share|cite|improve this question




share|cite|improve this question








edited Jan 15 at 14:31







MathOverview

















asked Jan 11 at 18:40









MathOverviewMathOverview

8,71243163




8,71243163












  • $begingroup$
    This is just because of the algebraic properties of $wedge$. Have you tried writing it out explicitly with $alpha = dx^1$ and $beta=dx^2$ for starters?
    $endgroup$
    – Ted Shifrin
    Jan 11 at 18:45










  • $begingroup$
    @TedShifrin In this case supose $alpha, beta bigwedge^1(mathbb{R}^2)$. We will have $alpha= dx^1=g(y_1,y_2)dy^1$, $beta= dx^2=h(y_1,y_2)dy^2$, $alphawedge beta(e_1,e_2)=1$ and $alphawedge beta g(y)h(y)det begin{pmatrix} dy^1(e_1) & dy^1(e_2) \ dy^2(e_1) & dy^2(e_2) end{pmatrix}$. But how can I use this?
    $endgroup$
    – MathOverview
    Jan 11 at 20:12












  • $begingroup$
    First, it's just about pointwise linear algebra. Second, don't do it by evaluating. Write $dy^i = sum a^i_j dx^j$. (In your set-up, you did the reverse. It doesn't matter.)
    $endgroup$
    – Ted Shifrin
    Jan 11 at 21:30










  • $begingroup$
    @TedShifrin It would be something of the type $dy^{I}wedge dy^{J}=dy^{i_1}wedge ldots wedge dy^{i_k}wedge dy^{j_1}wedge ldots wedge dy^{j_ell}=left( sum_{u=1}^{n} a_u^{i_1} dx^u right)wedge ldotswedge left( sum_{u=1}^{n} a_u^{i_k} dx^u right)wedge left( sum_{v=1}^{n} a_v^{j_1} dx^v right) wedge ldotswedge left( sum_{v=1}^{n} a_v^{j_ell} dx^v right);;;;?$
    $endgroup$
    – MathOverview
    Jan 11 at 21:50




















  • $begingroup$
    This is just because of the algebraic properties of $wedge$. Have you tried writing it out explicitly with $alpha = dx^1$ and $beta=dx^2$ for starters?
    $endgroup$
    – Ted Shifrin
    Jan 11 at 18:45










  • $begingroup$
    @TedShifrin In this case supose $alpha, beta bigwedge^1(mathbb{R}^2)$. We will have $alpha= dx^1=g(y_1,y_2)dy^1$, $beta= dx^2=h(y_1,y_2)dy^2$, $alphawedge beta(e_1,e_2)=1$ and $alphawedge beta g(y)h(y)det begin{pmatrix} dy^1(e_1) & dy^1(e_2) \ dy^2(e_1) & dy^2(e_2) end{pmatrix}$. But how can I use this?
    $endgroup$
    – MathOverview
    Jan 11 at 20:12












  • $begingroup$
    First, it's just about pointwise linear algebra. Second, don't do it by evaluating. Write $dy^i = sum a^i_j dx^j$. (In your set-up, you did the reverse. It doesn't matter.)
    $endgroup$
    – Ted Shifrin
    Jan 11 at 21:30










  • $begingroup$
    @TedShifrin It would be something of the type $dy^{I}wedge dy^{J}=dy^{i_1}wedge ldots wedge dy^{i_k}wedge dy^{j_1}wedge ldots wedge dy^{j_ell}=left( sum_{u=1}^{n} a_u^{i_1} dx^u right)wedge ldotswedge left( sum_{u=1}^{n} a_u^{i_k} dx^u right)wedge left( sum_{v=1}^{n} a_v^{j_1} dx^v right) wedge ldotswedge left( sum_{v=1}^{n} a_v^{j_ell} dx^v right);;;;?$
    $endgroup$
    – MathOverview
    Jan 11 at 21:50


















$begingroup$
This is just because of the algebraic properties of $wedge$. Have you tried writing it out explicitly with $alpha = dx^1$ and $beta=dx^2$ for starters?
$endgroup$
– Ted Shifrin
Jan 11 at 18:45




$begingroup$
This is just because of the algebraic properties of $wedge$. Have you tried writing it out explicitly with $alpha = dx^1$ and $beta=dx^2$ for starters?
$endgroup$
– Ted Shifrin
Jan 11 at 18:45












$begingroup$
@TedShifrin In this case supose $alpha, beta bigwedge^1(mathbb{R}^2)$. We will have $alpha= dx^1=g(y_1,y_2)dy^1$, $beta= dx^2=h(y_1,y_2)dy^2$, $alphawedge beta(e_1,e_2)=1$ and $alphawedge beta g(y)h(y)det begin{pmatrix} dy^1(e_1) & dy^1(e_2) \ dy^2(e_1) & dy^2(e_2) end{pmatrix}$. But how can I use this?
$endgroup$
– MathOverview
Jan 11 at 20:12






$begingroup$
@TedShifrin In this case supose $alpha, beta bigwedge^1(mathbb{R}^2)$. We will have $alpha= dx^1=g(y_1,y_2)dy^1$, $beta= dx^2=h(y_1,y_2)dy^2$, $alphawedge beta(e_1,e_2)=1$ and $alphawedge beta g(y)h(y)det begin{pmatrix} dy^1(e_1) & dy^1(e_2) \ dy^2(e_1) & dy^2(e_2) end{pmatrix}$. But how can I use this?
$endgroup$
– MathOverview
Jan 11 at 20:12














$begingroup$
First, it's just about pointwise linear algebra. Second, don't do it by evaluating. Write $dy^i = sum a^i_j dx^j$. (In your set-up, you did the reverse. It doesn't matter.)
$endgroup$
– Ted Shifrin
Jan 11 at 21:30




$begingroup$
First, it's just about pointwise linear algebra. Second, don't do it by evaluating. Write $dy^i = sum a^i_j dx^j$. (In your set-up, you did the reverse. It doesn't matter.)
$endgroup$
– Ted Shifrin
Jan 11 at 21:30












$begingroup$
@TedShifrin It would be something of the type $dy^{I}wedge dy^{J}=dy^{i_1}wedge ldots wedge dy^{i_k}wedge dy^{j_1}wedge ldots wedge dy^{j_ell}=left( sum_{u=1}^{n} a_u^{i_1} dx^u right)wedge ldotswedge left( sum_{u=1}^{n} a_u^{i_k} dx^u right)wedge left( sum_{v=1}^{n} a_v^{j_1} dx^v right) wedge ldotswedge left( sum_{v=1}^{n} a_v^{j_ell} dx^v right);;;;?$
$endgroup$
– MathOverview
Jan 11 at 21:50






$begingroup$
@TedShifrin It would be something of the type $dy^{I}wedge dy^{J}=dy^{i_1}wedge ldots wedge dy^{i_k}wedge dy^{j_1}wedge ldots wedge dy^{j_ell}=left( sum_{u=1}^{n} a_u^{i_1} dx^u right)wedge ldotswedge left( sum_{u=1}^{n} a_u^{i_k} dx^u right)wedge left( sum_{v=1}^{n} a_v^{j_1} dx^v right) wedge ldotswedge left( sum_{v=1}^{n} a_v^{j_ell} dx^v right);;;;?$
$endgroup$
– MathOverview
Jan 11 at 21:50












1 Answer
1






active

oldest

votes


















0












$begingroup$

I do not know if that is the most elegant and direct answer to the question. I would appreciate suggestions for improving it as well as answers that explore a different point of view.




Lema. Let $omegain bigwedge^{r}(mathbb{R}^n)$. Supose that




  • $sum_{K}a_K du^K$ is the expression of $omega$ in the basis ${du^K}_{K}$ of $bigwedge^{r}(mathbb{R}^n)$ that we get ( by wedge product $du^K=du^{k_1}wedge ldotswedge du^{k_r}$) from basis ${du^1, ldots, du^n}$ of $(mathbb{R}^n)^ast$ dual of basis ${u_1,ldots,u_n}$ of $mathbb{R}^n$.


  • $sum_{L}b_L dv^L$ is the expression of $omega$ in the basis ${dv^L}_{L}$ of $bigwedge^{r}(mathbb{R}^n)$ that we get ( by wedge product $dv^L=dv^{ell_1}wedge ldotswedge dv^{ell_r}$) from basis ${dv^1, ldots, dv^n}$ of $(mathbb{R}^n)^ast$ dual of canonical basis ${v_1,ldots,v_n}$ of $mathbb{R}^n$.


  • $c=(c_{kell})_{mtimes m}$ is the basis change matrix that expresses the basis ${u_1,ldots,u_n}$ in terms of the basis ${v_1,ldots,v_n}$ by the following equations $u_k=sum_{ell=1}^{m}c_{kell}v_ell$.



Then
$$
a_K=sum_{I} b_L det(c_{k_qell_p})_{rtimes r}
$$

with $(c_{k_qell_p})_{rtimes r}$ the matrix $r times r$ whose elements are the elenents of matrix $c=(c_{kell})$ whose indices are such that $k_qin K={k_1<ldots<k_r}$ and $ell_pin L={ell_1<ldots<j_r}$.




Proof. Fix $K={k_1<ldots<k_r}$ and set $u_{k_1},ldots, u_{k_p},ldots u_{k_r}in{u_1,ldots, u_n}$. We have
$$
omega(u_{k_1},ldots, u_{k_p},ldots u_{k_r})
=
sum_{I}a_Idu^I(u_{k_1},ldots, u_{k_p},ldots u_{k_r})
=
a_K
$$

On the other hand,
begin{align}
omega(u_{k_1},ldots, u_{k_p},ldots u_{k_r})
=&
sum_{L}b_Ldv^I(u_{k_1},ldots, u_{k_p},ldots u_{k_r})
\
=&
sum_{ell_1<ldots<ell_q<ldots <ell_r}
b_{{ell_1<ldots<ell_q<ldots <ell_r}}
dv^{ell_1}wedgeldotswedge dv^{ell_q}wedge ldots wedge du^{ell_r}
(u_{k_1},ldots, u_{k_p},ldots u_{k_r})
\
=&
sum_{ell_1<ldots<ell_q<ldots <ell_r}
b_{{ell_1<ldots<ell_q<ldots <ell_r}}
det( dv^{ell_q}cdot u_{k_p})_{rtimes r}
\
=&
sum_{ell_1<ldots<ell_q<ldots <ell_r}
b_{{ell_1<ldots<ell_q<ldots <ell_r}}
detleft( dv^{ell_q}cdot left( sum_{ell=1}^{n}c_{k_pell}v_ell right)right)_{rtimes r}
\
=&
sum_{ell_1<ldots<ell_q<ldots <ell_r}
b_{{ell_1<ldots<ell_q<ldots <ell_r}}
detleft( c_{k_qell_p}cdot dv^{ell_q} v_{ell_q}right)_{rtimes r}
\
=&
sum_{ell_1<ldots<ell_q<ldots <ell_r}
b_{{ell_1<ldots<ell_q<ldots <ell_r}}
detleft( c_{k_qell_p}cdot 1 right)_{rtimes r}
\
=&
sum_{L}b_{L}
detleft( c_{k_qell_p} right)_{rtimes r}
\
end{align}

Therefore, it follows that
$$
a_K=sum_{I} b_L det(c_{k_qell_p})_{rtimes r}.
$$





Now the demonstration of the question that this post refers to. Since
$$
left( sum_{I}a_I dx^Iright) wedge left( sum_{J}b_J dx^Jright)in
bigwedge^{r+s}(mathbb{R}^n)
$$

there are numbers $a_{K}$, with $K$ running through all ordered sets ${k_1< ldots< k_{r+s}}subset {1,ldots, n}$, such that
$$
left( sum_{I}a_I dx^Iright) wedge left( sum_{J}b_J dx^Jright)
=
sum_{K}a_K dx^K
$$

because ${dx^K}_{K}$ is basis of $bigwedge^{r+s}(mathbb{R}^n)$.For the same reasons given above there are numbers $b_{L}$, with $L$ running through all ordered sets ${ell_1< ldots< ell_{r+s}}subset {1,ldots, n}$, such that
$$
left( sum_{I}g_I dy^I right) wedge left( sum_{J} h_{J} dy^Jright)
=
sum_{L}b_L dy^L.
$$

Fix $K={k_1<ldots< k_{r+s} }$ and $x_{k_1},ldots, x_{k_q}, ldots, x_{r+s}$. It is easy to see that
$$
left( sum_{I}a_I dx^Iright) wedge left( sum_{J}b_J dx^Jright)
(x_{k_1},ldots, x_{k_q}, ldots, x_{r+s})
=
sum_{G}a_G dx^G
(x_{k_1},ldots, x_{k_q}, ldots, x_{k_{r+s}})
=
a_K
$$

Let $c=(c_{kell})_{mtimes m}$ is the basis change matrix that expresses the base ${x_1,ldots,x_n}$ in terms of the base ${y_1,ldots,y_n}$ by the following equations $x_k=sum_{ell=1}^{m}c_{kell}y_ell$. By analogous calculations to what we did in the above demonstration we have
begin{align}
left( sum_{I}g_I dy^I right) wedge left( sum_{J} h_{J} dy^Jright)
(x_{k_1},ldots, x_{k_q}, ldots, x_{k_{r+s}})
=&
sum_{L}b_L dy^L(x_{k_1},ldots, x_{k_q}, ldots, x_{k_{r+s}})
\
=&
sum_{I} b_L det(c_{k_qell_p})_{rtimes r}
end{align}

By lema, we have $a_K=sum_{I} b_L det(c_{k_qell_p})_{rtimes r}$, and it follows that
$$
left( sum_{I}a_I dx^Iright) wedge left( sum_{J}b_J dx^Jright)
hspace{1cm} mbox{ and } hspace{1cm}
left(sum_{I}g_I dy^I right) wedge left( sum_{J} h_{J} dy^Jright)
$$

are equals in all $(r+s)$-tuple of vectors $x_{k_1},ldots, x_{k_q}, ldots, x_{k_{r+s}}$ in basis ${x_1,ldots,x_n}$. By linearity we have that equality holds for all $(r+s)$-tuple of vectors $w_{k_1},ldots, w_{k_q}, ldots, w_{k_{r+s}}$ in $mathbb{R}^n$.






share|cite|improve this answer











$endgroup$













    Your Answer





    StackExchange.ifUsing("editor", function () {
    return StackExchange.using("mathjaxEditing", function () {
    StackExchange.MarkdownEditor.creationCallbacks.add(function (editor, postfix) {
    StackExchange.mathjaxEditing.prepareWmdForMathJax(editor, postfix, [["$", "$"], ["\\(","\\)"]]);
    });
    });
    }, "mathjax-editing");

    StackExchange.ready(function() {
    var channelOptions = {
    tags: "".split(" "),
    id: "69"
    };
    initTagRenderer("".split(" "), "".split(" "), channelOptions);

    StackExchange.using("externalEditor", function() {
    // Have to fire editor after snippets, if snippets enabled
    if (StackExchange.settings.snippets.snippetsEnabled) {
    StackExchange.using("snippets", function() {
    createEditor();
    });
    }
    else {
    createEditor();
    }
    });

    function createEditor() {
    StackExchange.prepareEditor({
    heartbeatType: 'answer',
    autoActivateHeartbeat: false,
    convertImagesToLinks: true,
    noModals: true,
    showLowRepImageUploadWarning: true,
    reputationToPostImages: 10,
    bindNavPrevention: true,
    postfix: "",
    imageUploader: {
    brandingHtml: "Powered by u003ca class="icon-imgur-white" href="https://imgur.com/"u003eu003c/au003e",
    contentPolicyHtml: "User contributions licensed under u003ca href="https://creativecommons.org/licenses/by-sa/3.0/"u003ecc by-sa 3.0 with attribution requiredu003c/au003e u003ca href="https://stackoverflow.com/legal/content-policy"u003e(content policy)u003c/au003e",
    allowUrls: true
    },
    noCode: true, onDemand: true,
    discardSelector: ".discard-answer"
    ,immediatelyShowMarkdownHelp:true
    });


    }
    });














    draft saved

    draft discarded


















    StackExchange.ready(
    function () {
    StackExchange.openid.initPostLogin('.new-post-login', 'https%3a%2f%2fmath.stackexchange.com%2fquestions%2f3070200%2fhow-to-prove-that-the-definition-of-exterior-product-of-differential-forms-is-no%23new-answer', 'question_page');
    }
    );

    Post as a guest















    Required, but never shown

























    1 Answer
    1






    active

    oldest

    votes








    1 Answer
    1






    active

    oldest

    votes









    active

    oldest

    votes






    active

    oldest

    votes









    0












    $begingroup$

    I do not know if that is the most elegant and direct answer to the question. I would appreciate suggestions for improving it as well as answers that explore a different point of view.




    Lema. Let $omegain bigwedge^{r}(mathbb{R}^n)$. Supose that




    • $sum_{K}a_K du^K$ is the expression of $omega$ in the basis ${du^K}_{K}$ of $bigwedge^{r}(mathbb{R}^n)$ that we get ( by wedge product $du^K=du^{k_1}wedge ldotswedge du^{k_r}$) from basis ${du^1, ldots, du^n}$ of $(mathbb{R}^n)^ast$ dual of basis ${u_1,ldots,u_n}$ of $mathbb{R}^n$.


    • $sum_{L}b_L dv^L$ is the expression of $omega$ in the basis ${dv^L}_{L}$ of $bigwedge^{r}(mathbb{R}^n)$ that we get ( by wedge product $dv^L=dv^{ell_1}wedge ldotswedge dv^{ell_r}$) from basis ${dv^1, ldots, dv^n}$ of $(mathbb{R}^n)^ast$ dual of canonical basis ${v_1,ldots,v_n}$ of $mathbb{R}^n$.


    • $c=(c_{kell})_{mtimes m}$ is the basis change matrix that expresses the basis ${u_1,ldots,u_n}$ in terms of the basis ${v_1,ldots,v_n}$ by the following equations $u_k=sum_{ell=1}^{m}c_{kell}v_ell$.



    Then
    $$
    a_K=sum_{I} b_L det(c_{k_qell_p})_{rtimes r}
    $$

    with $(c_{k_qell_p})_{rtimes r}$ the matrix $r times r$ whose elements are the elenents of matrix $c=(c_{kell})$ whose indices are such that $k_qin K={k_1<ldots<k_r}$ and $ell_pin L={ell_1<ldots<j_r}$.




    Proof. Fix $K={k_1<ldots<k_r}$ and set $u_{k_1},ldots, u_{k_p},ldots u_{k_r}in{u_1,ldots, u_n}$. We have
    $$
    omega(u_{k_1},ldots, u_{k_p},ldots u_{k_r})
    =
    sum_{I}a_Idu^I(u_{k_1},ldots, u_{k_p},ldots u_{k_r})
    =
    a_K
    $$

    On the other hand,
    begin{align}
    omega(u_{k_1},ldots, u_{k_p},ldots u_{k_r})
    =&
    sum_{L}b_Ldv^I(u_{k_1},ldots, u_{k_p},ldots u_{k_r})
    \
    =&
    sum_{ell_1<ldots<ell_q<ldots <ell_r}
    b_{{ell_1<ldots<ell_q<ldots <ell_r}}
    dv^{ell_1}wedgeldotswedge dv^{ell_q}wedge ldots wedge du^{ell_r}
    (u_{k_1},ldots, u_{k_p},ldots u_{k_r})
    \
    =&
    sum_{ell_1<ldots<ell_q<ldots <ell_r}
    b_{{ell_1<ldots<ell_q<ldots <ell_r}}
    det( dv^{ell_q}cdot u_{k_p})_{rtimes r}
    \
    =&
    sum_{ell_1<ldots<ell_q<ldots <ell_r}
    b_{{ell_1<ldots<ell_q<ldots <ell_r}}
    detleft( dv^{ell_q}cdot left( sum_{ell=1}^{n}c_{k_pell}v_ell right)right)_{rtimes r}
    \
    =&
    sum_{ell_1<ldots<ell_q<ldots <ell_r}
    b_{{ell_1<ldots<ell_q<ldots <ell_r}}
    detleft( c_{k_qell_p}cdot dv^{ell_q} v_{ell_q}right)_{rtimes r}
    \
    =&
    sum_{ell_1<ldots<ell_q<ldots <ell_r}
    b_{{ell_1<ldots<ell_q<ldots <ell_r}}
    detleft( c_{k_qell_p}cdot 1 right)_{rtimes r}
    \
    =&
    sum_{L}b_{L}
    detleft( c_{k_qell_p} right)_{rtimes r}
    \
    end{align}

    Therefore, it follows that
    $$
    a_K=sum_{I} b_L det(c_{k_qell_p})_{rtimes r}.
    $$





    Now the demonstration of the question that this post refers to. Since
    $$
    left( sum_{I}a_I dx^Iright) wedge left( sum_{J}b_J dx^Jright)in
    bigwedge^{r+s}(mathbb{R}^n)
    $$

    there are numbers $a_{K}$, with $K$ running through all ordered sets ${k_1< ldots< k_{r+s}}subset {1,ldots, n}$, such that
    $$
    left( sum_{I}a_I dx^Iright) wedge left( sum_{J}b_J dx^Jright)
    =
    sum_{K}a_K dx^K
    $$

    because ${dx^K}_{K}$ is basis of $bigwedge^{r+s}(mathbb{R}^n)$.For the same reasons given above there are numbers $b_{L}$, with $L$ running through all ordered sets ${ell_1< ldots< ell_{r+s}}subset {1,ldots, n}$, such that
    $$
    left( sum_{I}g_I dy^I right) wedge left( sum_{J} h_{J} dy^Jright)
    =
    sum_{L}b_L dy^L.
    $$

    Fix $K={k_1<ldots< k_{r+s} }$ and $x_{k_1},ldots, x_{k_q}, ldots, x_{r+s}$. It is easy to see that
    $$
    left( sum_{I}a_I dx^Iright) wedge left( sum_{J}b_J dx^Jright)
    (x_{k_1},ldots, x_{k_q}, ldots, x_{r+s})
    =
    sum_{G}a_G dx^G
    (x_{k_1},ldots, x_{k_q}, ldots, x_{k_{r+s}})
    =
    a_K
    $$

    Let $c=(c_{kell})_{mtimes m}$ is the basis change matrix that expresses the base ${x_1,ldots,x_n}$ in terms of the base ${y_1,ldots,y_n}$ by the following equations $x_k=sum_{ell=1}^{m}c_{kell}y_ell$. By analogous calculations to what we did in the above demonstration we have
    begin{align}
    left( sum_{I}g_I dy^I right) wedge left( sum_{J} h_{J} dy^Jright)
    (x_{k_1},ldots, x_{k_q}, ldots, x_{k_{r+s}})
    =&
    sum_{L}b_L dy^L(x_{k_1},ldots, x_{k_q}, ldots, x_{k_{r+s}})
    \
    =&
    sum_{I} b_L det(c_{k_qell_p})_{rtimes r}
    end{align}

    By lema, we have $a_K=sum_{I} b_L det(c_{k_qell_p})_{rtimes r}$, and it follows that
    $$
    left( sum_{I}a_I dx^Iright) wedge left( sum_{J}b_J dx^Jright)
    hspace{1cm} mbox{ and } hspace{1cm}
    left(sum_{I}g_I dy^I right) wedge left( sum_{J} h_{J} dy^Jright)
    $$

    are equals in all $(r+s)$-tuple of vectors $x_{k_1},ldots, x_{k_q}, ldots, x_{k_{r+s}}$ in basis ${x_1,ldots,x_n}$. By linearity we have that equality holds for all $(r+s)$-tuple of vectors $w_{k_1},ldots, w_{k_q}, ldots, w_{k_{r+s}}$ in $mathbb{R}^n$.






    share|cite|improve this answer











    $endgroup$


















      0












      $begingroup$

      I do not know if that is the most elegant and direct answer to the question. I would appreciate suggestions for improving it as well as answers that explore a different point of view.




      Lema. Let $omegain bigwedge^{r}(mathbb{R}^n)$. Supose that




      • $sum_{K}a_K du^K$ is the expression of $omega$ in the basis ${du^K}_{K}$ of $bigwedge^{r}(mathbb{R}^n)$ that we get ( by wedge product $du^K=du^{k_1}wedge ldotswedge du^{k_r}$) from basis ${du^1, ldots, du^n}$ of $(mathbb{R}^n)^ast$ dual of basis ${u_1,ldots,u_n}$ of $mathbb{R}^n$.


      • $sum_{L}b_L dv^L$ is the expression of $omega$ in the basis ${dv^L}_{L}$ of $bigwedge^{r}(mathbb{R}^n)$ that we get ( by wedge product $dv^L=dv^{ell_1}wedge ldotswedge dv^{ell_r}$) from basis ${dv^1, ldots, dv^n}$ of $(mathbb{R}^n)^ast$ dual of canonical basis ${v_1,ldots,v_n}$ of $mathbb{R}^n$.


      • $c=(c_{kell})_{mtimes m}$ is the basis change matrix that expresses the basis ${u_1,ldots,u_n}$ in terms of the basis ${v_1,ldots,v_n}$ by the following equations $u_k=sum_{ell=1}^{m}c_{kell}v_ell$.



      Then
      $$
      a_K=sum_{I} b_L det(c_{k_qell_p})_{rtimes r}
      $$

      with $(c_{k_qell_p})_{rtimes r}$ the matrix $r times r$ whose elements are the elenents of matrix $c=(c_{kell})$ whose indices are such that $k_qin K={k_1<ldots<k_r}$ and $ell_pin L={ell_1<ldots<j_r}$.




      Proof. Fix $K={k_1<ldots<k_r}$ and set $u_{k_1},ldots, u_{k_p},ldots u_{k_r}in{u_1,ldots, u_n}$. We have
      $$
      omega(u_{k_1},ldots, u_{k_p},ldots u_{k_r})
      =
      sum_{I}a_Idu^I(u_{k_1},ldots, u_{k_p},ldots u_{k_r})
      =
      a_K
      $$

      On the other hand,
      begin{align}
      omega(u_{k_1},ldots, u_{k_p},ldots u_{k_r})
      =&
      sum_{L}b_Ldv^I(u_{k_1},ldots, u_{k_p},ldots u_{k_r})
      \
      =&
      sum_{ell_1<ldots<ell_q<ldots <ell_r}
      b_{{ell_1<ldots<ell_q<ldots <ell_r}}
      dv^{ell_1}wedgeldotswedge dv^{ell_q}wedge ldots wedge du^{ell_r}
      (u_{k_1},ldots, u_{k_p},ldots u_{k_r})
      \
      =&
      sum_{ell_1<ldots<ell_q<ldots <ell_r}
      b_{{ell_1<ldots<ell_q<ldots <ell_r}}
      det( dv^{ell_q}cdot u_{k_p})_{rtimes r}
      \
      =&
      sum_{ell_1<ldots<ell_q<ldots <ell_r}
      b_{{ell_1<ldots<ell_q<ldots <ell_r}}
      detleft( dv^{ell_q}cdot left( sum_{ell=1}^{n}c_{k_pell}v_ell right)right)_{rtimes r}
      \
      =&
      sum_{ell_1<ldots<ell_q<ldots <ell_r}
      b_{{ell_1<ldots<ell_q<ldots <ell_r}}
      detleft( c_{k_qell_p}cdot dv^{ell_q} v_{ell_q}right)_{rtimes r}
      \
      =&
      sum_{ell_1<ldots<ell_q<ldots <ell_r}
      b_{{ell_1<ldots<ell_q<ldots <ell_r}}
      detleft( c_{k_qell_p}cdot 1 right)_{rtimes r}
      \
      =&
      sum_{L}b_{L}
      detleft( c_{k_qell_p} right)_{rtimes r}
      \
      end{align}

      Therefore, it follows that
      $$
      a_K=sum_{I} b_L det(c_{k_qell_p})_{rtimes r}.
      $$





      Now the demonstration of the question that this post refers to. Since
      $$
      left( sum_{I}a_I dx^Iright) wedge left( sum_{J}b_J dx^Jright)in
      bigwedge^{r+s}(mathbb{R}^n)
      $$

      there are numbers $a_{K}$, with $K$ running through all ordered sets ${k_1< ldots< k_{r+s}}subset {1,ldots, n}$, such that
      $$
      left( sum_{I}a_I dx^Iright) wedge left( sum_{J}b_J dx^Jright)
      =
      sum_{K}a_K dx^K
      $$

      because ${dx^K}_{K}$ is basis of $bigwedge^{r+s}(mathbb{R}^n)$.For the same reasons given above there are numbers $b_{L}$, with $L$ running through all ordered sets ${ell_1< ldots< ell_{r+s}}subset {1,ldots, n}$, such that
      $$
      left( sum_{I}g_I dy^I right) wedge left( sum_{J} h_{J} dy^Jright)
      =
      sum_{L}b_L dy^L.
      $$

      Fix $K={k_1<ldots< k_{r+s} }$ and $x_{k_1},ldots, x_{k_q}, ldots, x_{r+s}$. It is easy to see that
      $$
      left( sum_{I}a_I dx^Iright) wedge left( sum_{J}b_J dx^Jright)
      (x_{k_1},ldots, x_{k_q}, ldots, x_{r+s})
      =
      sum_{G}a_G dx^G
      (x_{k_1},ldots, x_{k_q}, ldots, x_{k_{r+s}})
      =
      a_K
      $$

      Let $c=(c_{kell})_{mtimes m}$ is the basis change matrix that expresses the base ${x_1,ldots,x_n}$ in terms of the base ${y_1,ldots,y_n}$ by the following equations $x_k=sum_{ell=1}^{m}c_{kell}y_ell$. By analogous calculations to what we did in the above demonstration we have
      begin{align}
      left( sum_{I}g_I dy^I right) wedge left( sum_{J} h_{J} dy^Jright)
      (x_{k_1},ldots, x_{k_q}, ldots, x_{k_{r+s}})
      =&
      sum_{L}b_L dy^L(x_{k_1},ldots, x_{k_q}, ldots, x_{k_{r+s}})
      \
      =&
      sum_{I} b_L det(c_{k_qell_p})_{rtimes r}
      end{align}

      By lema, we have $a_K=sum_{I} b_L det(c_{k_qell_p})_{rtimes r}$, and it follows that
      $$
      left( sum_{I}a_I dx^Iright) wedge left( sum_{J}b_J dx^Jright)
      hspace{1cm} mbox{ and } hspace{1cm}
      left(sum_{I}g_I dy^I right) wedge left( sum_{J} h_{J} dy^Jright)
      $$

      are equals in all $(r+s)$-tuple of vectors $x_{k_1},ldots, x_{k_q}, ldots, x_{k_{r+s}}$ in basis ${x_1,ldots,x_n}$. By linearity we have that equality holds for all $(r+s)$-tuple of vectors $w_{k_1},ldots, w_{k_q}, ldots, w_{k_{r+s}}$ in $mathbb{R}^n$.






      share|cite|improve this answer











      $endgroup$
















        0












        0








        0





        $begingroup$

        I do not know if that is the most elegant and direct answer to the question. I would appreciate suggestions for improving it as well as answers that explore a different point of view.




        Lema. Let $omegain bigwedge^{r}(mathbb{R}^n)$. Supose that




        • $sum_{K}a_K du^K$ is the expression of $omega$ in the basis ${du^K}_{K}$ of $bigwedge^{r}(mathbb{R}^n)$ that we get ( by wedge product $du^K=du^{k_1}wedge ldotswedge du^{k_r}$) from basis ${du^1, ldots, du^n}$ of $(mathbb{R}^n)^ast$ dual of basis ${u_1,ldots,u_n}$ of $mathbb{R}^n$.


        • $sum_{L}b_L dv^L$ is the expression of $omega$ in the basis ${dv^L}_{L}$ of $bigwedge^{r}(mathbb{R}^n)$ that we get ( by wedge product $dv^L=dv^{ell_1}wedge ldotswedge dv^{ell_r}$) from basis ${dv^1, ldots, dv^n}$ of $(mathbb{R}^n)^ast$ dual of canonical basis ${v_1,ldots,v_n}$ of $mathbb{R}^n$.


        • $c=(c_{kell})_{mtimes m}$ is the basis change matrix that expresses the basis ${u_1,ldots,u_n}$ in terms of the basis ${v_1,ldots,v_n}$ by the following equations $u_k=sum_{ell=1}^{m}c_{kell}v_ell$.



        Then
        $$
        a_K=sum_{I} b_L det(c_{k_qell_p})_{rtimes r}
        $$

        with $(c_{k_qell_p})_{rtimes r}$ the matrix $r times r$ whose elements are the elenents of matrix $c=(c_{kell})$ whose indices are such that $k_qin K={k_1<ldots<k_r}$ and $ell_pin L={ell_1<ldots<j_r}$.




        Proof. Fix $K={k_1<ldots<k_r}$ and set $u_{k_1},ldots, u_{k_p},ldots u_{k_r}in{u_1,ldots, u_n}$. We have
        $$
        omega(u_{k_1},ldots, u_{k_p},ldots u_{k_r})
        =
        sum_{I}a_Idu^I(u_{k_1},ldots, u_{k_p},ldots u_{k_r})
        =
        a_K
        $$

        On the other hand,
        begin{align}
        omega(u_{k_1},ldots, u_{k_p},ldots u_{k_r})
        =&
        sum_{L}b_Ldv^I(u_{k_1},ldots, u_{k_p},ldots u_{k_r})
        \
        =&
        sum_{ell_1<ldots<ell_q<ldots <ell_r}
        b_{{ell_1<ldots<ell_q<ldots <ell_r}}
        dv^{ell_1}wedgeldotswedge dv^{ell_q}wedge ldots wedge du^{ell_r}
        (u_{k_1},ldots, u_{k_p},ldots u_{k_r})
        \
        =&
        sum_{ell_1<ldots<ell_q<ldots <ell_r}
        b_{{ell_1<ldots<ell_q<ldots <ell_r}}
        det( dv^{ell_q}cdot u_{k_p})_{rtimes r}
        \
        =&
        sum_{ell_1<ldots<ell_q<ldots <ell_r}
        b_{{ell_1<ldots<ell_q<ldots <ell_r}}
        detleft( dv^{ell_q}cdot left( sum_{ell=1}^{n}c_{k_pell}v_ell right)right)_{rtimes r}
        \
        =&
        sum_{ell_1<ldots<ell_q<ldots <ell_r}
        b_{{ell_1<ldots<ell_q<ldots <ell_r}}
        detleft( c_{k_qell_p}cdot dv^{ell_q} v_{ell_q}right)_{rtimes r}
        \
        =&
        sum_{ell_1<ldots<ell_q<ldots <ell_r}
        b_{{ell_1<ldots<ell_q<ldots <ell_r}}
        detleft( c_{k_qell_p}cdot 1 right)_{rtimes r}
        \
        =&
        sum_{L}b_{L}
        detleft( c_{k_qell_p} right)_{rtimes r}
        \
        end{align}

        Therefore, it follows that
        $$
        a_K=sum_{I} b_L det(c_{k_qell_p})_{rtimes r}.
        $$





        Now the demonstration of the question that this post refers to. Since
        $$
        left( sum_{I}a_I dx^Iright) wedge left( sum_{J}b_J dx^Jright)in
        bigwedge^{r+s}(mathbb{R}^n)
        $$

        there are numbers $a_{K}$, with $K$ running through all ordered sets ${k_1< ldots< k_{r+s}}subset {1,ldots, n}$, such that
        $$
        left( sum_{I}a_I dx^Iright) wedge left( sum_{J}b_J dx^Jright)
        =
        sum_{K}a_K dx^K
        $$

        because ${dx^K}_{K}$ is basis of $bigwedge^{r+s}(mathbb{R}^n)$.For the same reasons given above there are numbers $b_{L}$, with $L$ running through all ordered sets ${ell_1< ldots< ell_{r+s}}subset {1,ldots, n}$, such that
        $$
        left( sum_{I}g_I dy^I right) wedge left( sum_{J} h_{J} dy^Jright)
        =
        sum_{L}b_L dy^L.
        $$

        Fix $K={k_1<ldots< k_{r+s} }$ and $x_{k_1},ldots, x_{k_q}, ldots, x_{r+s}$. It is easy to see that
        $$
        left( sum_{I}a_I dx^Iright) wedge left( sum_{J}b_J dx^Jright)
        (x_{k_1},ldots, x_{k_q}, ldots, x_{r+s})
        =
        sum_{G}a_G dx^G
        (x_{k_1},ldots, x_{k_q}, ldots, x_{k_{r+s}})
        =
        a_K
        $$

        Let $c=(c_{kell})_{mtimes m}$ is the basis change matrix that expresses the base ${x_1,ldots,x_n}$ in terms of the base ${y_1,ldots,y_n}$ by the following equations $x_k=sum_{ell=1}^{m}c_{kell}y_ell$. By analogous calculations to what we did in the above demonstration we have
        begin{align}
        left( sum_{I}g_I dy^I right) wedge left( sum_{J} h_{J} dy^Jright)
        (x_{k_1},ldots, x_{k_q}, ldots, x_{k_{r+s}})
        =&
        sum_{L}b_L dy^L(x_{k_1},ldots, x_{k_q}, ldots, x_{k_{r+s}})
        \
        =&
        sum_{I} b_L det(c_{k_qell_p})_{rtimes r}
        end{align}

        By lema, we have $a_K=sum_{I} b_L det(c_{k_qell_p})_{rtimes r}$, and it follows that
        $$
        left( sum_{I}a_I dx^Iright) wedge left( sum_{J}b_J dx^Jright)
        hspace{1cm} mbox{ and } hspace{1cm}
        left(sum_{I}g_I dy^I right) wedge left( sum_{J} h_{J} dy^Jright)
        $$

        are equals in all $(r+s)$-tuple of vectors $x_{k_1},ldots, x_{k_q}, ldots, x_{k_{r+s}}$ in basis ${x_1,ldots,x_n}$. By linearity we have that equality holds for all $(r+s)$-tuple of vectors $w_{k_1},ldots, w_{k_q}, ldots, w_{k_{r+s}}$ in $mathbb{R}^n$.






        share|cite|improve this answer











        $endgroup$



        I do not know if that is the most elegant and direct answer to the question. I would appreciate suggestions for improving it as well as answers that explore a different point of view.




        Lema. Let $omegain bigwedge^{r}(mathbb{R}^n)$. Supose that




        • $sum_{K}a_K du^K$ is the expression of $omega$ in the basis ${du^K}_{K}$ of $bigwedge^{r}(mathbb{R}^n)$ that we get ( by wedge product $du^K=du^{k_1}wedge ldotswedge du^{k_r}$) from basis ${du^1, ldots, du^n}$ of $(mathbb{R}^n)^ast$ dual of basis ${u_1,ldots,u_n}$ of $mathbb{R}^n$.


        • $sum_{L}b_L dv^L$ is the expression of $omega$ in the basis ${dv^L}_{L}$ of $bigwedge^{r}(mathbb{R}^n)$ that we get ( by wedge product $dv^L=dv^{ell_1}wedge ldotswedge dv^{ell_r}$) from basis ${dv^1, ldots, dv^n}$ of $(mathbb{R}^n)^ast$ dual of canonical basis ${v_1,ldots,v_n}$ of $mathbb{R}^n$.


        • $c=(c_{kell})_{mtimes m}$ is the basis change matrix that expresses the basis ${u_1,ldots,u_n}$ in terms of the basis ${v_1,ldots,v_n}$ by the following equations $u_k=sum_{ell=1}^{m}c_{kell}v_ell$.



        Then
        $$
        a_K=sum_{I} b_L det(c_{k_qell_p})_{rtimes r}
        $$

        with $(c_{k_qell_p})_{rtimes r}$ the matrix $r times r$ whose elements are the elenents of matrix $c=(c_{kell})$ whose indices are such that $k_qin K={k_1<ldots<k_r}$ and $ell_pin L={ell_1<ldots<j_r}$.




        Proof. Fix $K={k_1<ldots<k_r}$ and set $u_{k_1},ldots, u_{k_p},ldots u_{k_r}in{u_1,ldots, u_n}$. We have
        $$
        omega(u_{k_1},ldots, u_{k_p},ldots u_{k_r})
        =
        sum_{I}a_Idu^I(u_{k_1},ldots, u_{k_p},ldots u_{k_r})
        =
        a_K
        $$

        On the other hand,
        begin{align}
        omega(u_{k_1},ldots, u_{k_p},ldots u_{k_r})
        =&
        sum_{L}b_Ldv^I(u_{k_1},ldots, u_{k_p},ldots u_{k_r})
        \
        =&
        sum_{ell_1<ldots<ell_q<ldots <ell_r}
        b_{{ell_1<ldots<ell_q<ldots <ell_r}}
        dv^{ell_1}wedgeldotswedge dv^{ell_q}wedge ldots wedge du^{ell_r}
        (u_{k_1},ldots, u_{k_p},ldots u_{k_r})
        \
        =&
        sum_{ell_1<ldots<ell_q<ldots <ell_r}
        b_{{ell_1<ldots<ell_q<ldots <ell_r}}
        det( dv^{ell_q}cdot u_{k_p})_{rtimes r}
        \
        =&
        sum_{ell_1<ldots<ell_q<ldots <ell_r}
        b_{{ell_1<ldots<ell_q<ldots <ell_r}}
        detleft( dv^{ell_q}cdot left( sum_{ell=1}^{n}c_{k_pell}v_ell right)right)_{rtimes r}
        \
        =&
        sum_{ell_1<ldots<ell_q<ldots <ell_r}
        b_{{ell_1<ldots<ell_q<ldots <ell_r}}
        detleft( c_{k_qell_p}cdot dv^{ell_q} v_{ell_q}right)_{rtimes r}
        \
        =&
        sum_{ell_1<ldots<ell_q<ldots <ell_r}
        b_{{ell_1<ldots<ell_q<ldots <ell_r}}
        detleft( c_{k_qell_p}cdot 1 right)_{rtimes r}
        \
        =&
        sum_{L}b_{L}
        detleft( c_{k_qell_p} right)_{rtimes r}
        \
        end{align}

        Therefore, it follows that
        $$
        a_K=sum_{I} b_L det(c_{k_qell_p})_{rtimes r}.
        $$





        Now the demonstration of the question that this post refers to. Since
        $$
        left( sum_{I}a_I dx^Iright) wedge left( sum_{J}b_J dx^Jright)in
        bigwedge^{r+s}(mathbb{R}^n)
        $$

        there are numbers $a_{K}$, with $K$ running through all ordered sets ${k_1< ldots< k_{r+s}}subset {1,ldots, n}$, such that
        $$
        left( sum_{I}a_I dx^Iright) wedge left( sum_{J}b_J dx^Jright)
        =
        sum_{K}a_K dx^K
        $$

        because ${dx^K}_{K}$ is basis of $bigwedge^{r+s}(mathbb{R}^n)$.For the same reasons given above there are numbers $b_{L}$, with $L$ running through all ordered sets ${ell_1< ldots< ell_{r+s}}subset {1,ldots, n}$, such that
        $$
        left( sum_{I}g_I dy^I right) wedge left( sum_{J} h_{J} dy^Jright)
        =
        sum_{L}b_L dy^L.
        $$

        Fix $K={k_1<ldots< k_{r+s} }$ and $x_{k_1},ldots, x_{k_q}, ldots, x_{r+s}$. It is easy to see that
        $$
        left( sum_{I}a_I dx^Iright) wedge left( sum_{J}b_J dx^Jright)
        (x_{k_1},ldots, x_{k_q}, ldots, x_{r+s})
        =
        sum_{G}a_G dx^G
        (x_{k_1},ldots, x_{k_q}, ldots, x_{k_{r+s}})
        =
        a_K
        $$

        Let $c=(c_{kell})_{mtimes m}$ is the basis change matrix that expresses the base ${x_1,ldots,x_n}$ in terms of the base ${y_1,ldots,y_n}$ by the following equations $x_k=sum_{ell=1}^{m}c_{kell}y_ell$. By analogous calculations to what we did in the above demonstration we have
        begin{align}
        left( sum_{I}g_I dy^I right) wedge left( sum_{J} h_{J} dy^Jright)
        (x_{k_1},ldots, x_{k_q}, ldots, x_{k_{r+s}})
        =&
        sum_{L}b_L dy^L(x_{k_1},ldots, x_{k_q}, ldots, x_{k_{r+s}})
        \
        =&
        sum_{I} b_L det(c_{k_qell_p})_{rtimes r}
        end{align}

        By lema, we have $a_K=sum_{I} b_L det(c_{k_qell_p})_{rtimes r}$, and it follows that
        $$
        left( sum_{I}a_I dx^Iright) wedge left( sum_{J}b_J dx^Jright)
        hspace{1cm} mbox{ and } hspace{1cm}
        left(sum_{I}g_I dy^I right) wedge left( sum_{J} h_{J} dy^Jright)
        $$

        are equals in all $(r+s)$-tuple of vectors $x_{k_1},ldots, x_{k_q}, ldots, x_{k_{r+s}}$ in basis ${x_1,ldots,x_n}$. By linearity we have that equality holds for all $(r+s)$-tuple of vectors $w_{k_1},ldots, w_{k_q}, ldots, w_{k_{r+s}}$ in $mathbb{R}^n$.







        share|cite|improve this answer














        share|cite|improve this answer



        share|cite|improve this answer








        edited Jan 15 at 17:19

























        answered Jan 15 at 16:10









        MathOverviewMathOverview

        8,71243163




        8,71243163






























            draft saved

            draft discarded




















































            Thanks for contributing an answer to Mathematics Stack Exchange!


            • Please be sure to answer the question. Provide details and share your research!

            But avoid



            • Asking for help, clarification, or responding to other answers.

            • Making statements based on opinion; back them up with references or personal experience.


            Use MathJax to format equations. MathJax reference.


            To learn more, see our tips on writing great answers.




            draft saved


            draft discarded














            StackExchange.ready(
            function () {
            StackExchange.openid.initPostLogin('.new-post-login', 'https%3a%2f%2fmath.stackexchange.com%2fquestions%2f3070200%2fhow-to-prove-that-the-definition-of-exterior-product-of-differential-forms-is-no%23new-answer', 'question_page');
            }
            );

            Post as a guest















            Required, but never shown





















































            Required, but never shown














            Required, but never shown












            Required, but never shown







            Required, but never shown

































            Required, but never shown














            Required, but never shown












            Required, but never shown







            Required, but never shown







            Popular posts from this blog

            Can a sorcerer learn a 5th-level spell early by creating spell slots using the Font of Magic feature?

            Does disintegrating a polymorphed enemy still kill it after the 2018 errata?

            A Topological Invariant for $pi_3(U(n))$